Difference between revisions of "2021 AMC 10B Problems/Problem 7"

m (Problem)
(Solution)
Line 4: Line 4:
 
<math>\textbf{(A) }24\pi \qquad \textbf{(B) }32\pi \qquad \textbf{(C) }64\pi \qquad \textbf{(D) }65\pi \qquad \textbf{(E) }84\pi</math>
 
<math>\textbf{(A) }24\pi \qquad \textbf{(B) }32\pi \qquad \textbf{(C) }64\pi \qquad \textbf{(D) }65\pi \qquad \textbf{(E) }84\pi</math>
 
==Solution==
 
==Solution==
D
+
[asy]
 +
pair A=(10,0);
 +
pair B=(-10,0);
 +
draw(A--B);
 +
draw(circle((0,-1),1));
 +
draw(circle((0,-3),3));
 +
draw(circle((0,-5),5));
 +
draw(circle((0,7),7));
 +
dot((0,7));
 +
draw((0,7)--(0,0));
 +
label("<math>7</math>",(0,3.5),E);
 +
label("<math>l</math>",(-9,0),S);
 +
[/asy]
 +
After a bit of wishful thinking and inspection, we find that the above configuration maximizes our area. <math>49 \pi + (25-9) \pi=65 \pi \rightarrow \boxed{D}</math>

Revision as of 19:53, 11 February 2021

Problem

In a plane, four circles with radii $1,3,5,$ and $7$ are tangent to line $l$ at the same point $A,$ but they may be on either side of $l$. Region $S$ consists of all the points that lie inside exactly one of the four circles. What is the maximum possible area of region $S$?

$\textbf{(A) }24\pi \qquad \textbf{(B) }32\pi \qquad \textbf{(C) }64\pi \qquad \textbf{(D) }65\pi \qquad \textbf{(E) }84\pi$

Solution

[asy] pair A=(10,0); pair B=(-10,0); draw(A--B); draw(circle((0,-1),1)); draw(circle((0,-3),3)); draw(circle((0,-5),5)); draw(circle((0,7),7)); dot((0,7)); draw((0,7)--(0,0)); label("$7$",(0,3.5),E); label("$l$",(-9,0),S); [/asy] After a bit of wishful thinking and inspection, we find that the above configuration maximizes our area. $49 \pi + (25-9) \pi=65 \pi \rightarrow \boxed{D}$